Wie quantisiert man den Phasenraum halbklassisch?

Wenn Leute Vorträge über semiklassische Theorien halten, sind sie oft sehr zwielichtig darüber, wie Quantisierung tatsächlich funktioniert.

Normalerweise beginnen sie damit, über eine Partition von zu sprechen -Zellen landen dann bei so etwas wie der WKB-Wellenfunktion und sprechen kurz darauf vom Limit 0 .

Die Menge, die quantisiert wird, ist normalerweise die Aktion p d q was ein halbes ganzes mal sein soll 2 π .

Über welche Kurve integrieren wir? Ist es eine Flugbahn, eine periodische Umlaufbahn oder so etwas? Und wie hängt das mit der Teilung in Planck-Zellen zusammen?

Außerdem, was ist die Bedeutung der Grenze 0 ?

Kapitel 2 dieser Vorlesung könnte hilfreich sein.
Für eine Erweiterung in (informativer als nur die klassische Grenze) siehe ns.math.cas.cz/~englis/38.ps
Es ist ein ganzzahliges Vielfaches 2 π , manchmal gibt es einen halbzahligen Offset. Die halbe ganze Zahl kommt vom HO-Grundzustand und hat eine halbklassische Erklärung in Bezug auf Maslov-Indizes, aber die ganzzahlige Folge ist die Hauptregel.

Antworten (3)

Zunächst muss man verstehen, dass der Phasenraum klassischerweise durch parametrisiert wird x , p und Koordinaten auf einer gewöhnlichen Ebene pendeln miteinander, x p = p x . In der Quantenmechanik ist dies jedoch nicht der Fall. Stattdessen haben wir den Heisenberg-Kommutator

x p p x = ich .
Das bedeutet, dass der Phasenraum quantenmechanisch keine gewöhnliche Ebene (oder höherdimensionaler Raum) ist, sondern eine "nichtkommutative Geometrie".

Dennoch kann man auch die gewöhnliche Ebene verwenden, um den Quantenphasenraum zu parametrisieren. Die Operatoren auf dem Hilbert-Raum können gewöhnlichen Funktionen kanonisch zugeordnet werden f ( x , p ) der Pendelkoordinaten über die sogenannte Wigner-Transformation:

http://en.wikipedia.org/wiki/Weyl_ordering

Es ist wichtig zu beachten, dass der Kommutator ungleich Null die Geometrie des Phasenraums nur beeinflusst, wenn wir ihn mit einer besseren Auflösung als untersuchen . Bei sehr großen Änderungen von x und p , kann der Quantenphasenraum durch den klassischen angenähert werden.

Das 0 ist die klassische Grenze. In dieser Grenze x p p x = ich wird ebenfalls auf Null gesetzt. Der Phasenraum wird also effektiv zu einer klassischen Geometrie. Wenn Sie behalten möchten fest und stellen Sie sich vor, was die Grenze bedeutet, es bedeutet solche Werte von x , p und ihre Änderungen damit Δ x Δ p . In dieser Grenze ist der Phasenraum klassisch und man kann darin gewöhnliche Konturintegrale definieren.

In dieser Grenze ist die Anzahl der in solchen Konturen eingeschlossenen "Zellen" riesig, viel größer als eine, N 1 . Man kann die Anzahl der Zellen berechnen (dh die Anzahl der Basisvektoren einer orthonormalen Basis, die ein Teilchen mit darstellen x , p in dem von der Kontur begrenzten Bereich) – mit einer Genauigkeit, die nur die „etwas unscharfe Grenze“ vernachlässigt – indem die Fläche genommen und durch dividiert wird 2 π .

Diese Karte kann als folgende Entsprechung zwischen der Quantenspur und dem klassischen Integral über den Phasenraum visualisiert werden:

T r 1 2 π d p d x
Diese Beziehung funktioniert für jeden Bereich im Phasenraum, der durch eine beliebige Kurve begrenzt ist. Aber es gibt spezielle Arten von Kurven, für die wir zusätzliche Dinge sagen können.

Wir können auch Beispiele geben, die erklären, warum die obige Korrespondenz funktioniert. Stellen Sie sich vor, dass der Raum ein Kreis mit einem Radius ist R , dh x identifiziert sich mit x + 2 π R , mit dem Umfang. In diesem Fall das Momentum p quantisiert ist, weil die Wellenfunktion exp ( ich p x / ) muss auf dem Kreis einwertig sein. Es folgt dem

p = N R
Die Eigenzustände von p bilden eine vollständige Basis und wir können zählen, wie viele Zustände es in einem Bereich des Phasenraums gibt. Stellen Sie sich den Phasenraum als dünnen Streifen vor (oder genauer gesagt als langen unendlichen Zylinder: wegen der periodischen Identifizierung von x ). Es ist dünn in der x Richtung, die Breite ist 2 π R , natürlich der Umfang des Kreises.

Wie viele Staaten gibt es? Wenn die Länge des Streifens/Zylinders in der p Richtung ist Δ p , die Anzahl der Zustände darin ist Δ p R / weil der Abstand ist / R . Und weil die Breite des Streifens ist 2 π R , die Anzahl der Zustände pro Flächeneinheit des Phasenraums ist

Δ p R 1 Δ p 1 2 π R = 1 2 π
was genau mit der Behauptung übereinstimmt, dass 2 π ist die Fläche der einzelnen Zelle. Dieses Ergebnis würde nicht nur für die Verdichtung auf dem Kreis gelten, sondern auch für das Unendliche x , p auch.

Die WKB-Näherung verwendet diese Erkenntnisse, um viele andere Dinge abzuleiten. Insbesondere die interessantesten Konturen für p d x in der WKB-Näherung sind Konturen Verbindungspunkte mit H ( x , p ) = H 0 , dh Konturen einer festen Energie. Das sind Orte des Phasenraums, an denen man sich einen Energie-Eigenzustand lokalisiert vorstellen kann. Wenn Sie viele Konturen dieser Form für verschiedene Werte von zeichnen H 0 und stellen Sie sicher, dass der Bereich dazwischen liegt 2 π für jedes benachbarte Paar können Sie behaupten, dass der „Kreisring“ zwischen den Konturen den Bereich des Phasenraums darstellt, der einem bestimmten Eigenzustand zugeordnet ist.

Sie können zum Beispiel den harmonischen Oszillator auswählen. In den richtigen Einheiten, die für das Diagramm ausgewählt wurden, H ( x , p ) = H 0 sind Kreise für jeden Wert von H 0 und die Bereiche zwischen den Konturen sind echte Ringe. Ihre Breite wird wie gehen 1 / n und ihr Radius wird wie gehen n wo n ist die Ganzzahl, die das Energieniveau bezeichnet. Sie sollten auch die den Ursprung umgebende Innenkontur so wählen, dass sie den entsprechenden Bereich des Phasenraums ebenfalls umgibt.

Die klassische Grenze ist nur dann genau genug, wenn der Abstand der Konturen dicht genug ist. Aber obwohl es dicht genug ist, kann man die tatsächliche Dichte quantifizieren – indem man nur die Methoden der Grenze anwendet. Um solche Dinge zu berechnen, verwenden wir wirklich nicht nur die "klassische Grenze"; Wir verwenden auch einige Informationen über die "erste Quantenkorrektur" der klassischen Physik.

Lassen Sie sich nicht einschüchtern, die semiklassische Quantisierung ist sehr einfach und kann anhand einiger Beispiele, die zum allgemeinen Fall führen, direkt verstanden werden.

Stellen Sie sich ein Teilchen in einer Kiste vor. Die klassischen Bewegungen sind Reflexionen von der Wand. Diese bilden eine Box im Phasenraum, wenn das Teilchen nach links geht, die Wand trifft, nach rechts geht und die andere Wand trifft. Wenn das Teilchen den Impuls p hat und die Länge des Kastens L ist, ist die von dieser Bewegung eingeschlossene Fläche im Phasenraum

p L

und die Bedingung ist, dass dies ein ganzzahliges Vielfaches von ist h = 2 π . Dies ergibt die Impulsquantisierungsbedingung aus der Quantenmechanik.

Für ein 1-dimensionales System gilt folgende Regel

p d x = n h

Mit einem möglichen Versatz, so dass die rechte Seite sein könnte ( n + 1 / 2 ) h , oder ( n + 3 / 4 ) h , wie angemessen, aber der Abstand zwischen den Ebenen wird durch diese Regel in führender Reihenfolge in h angegeben. Diese Regel kann aus der Beziehung von deBroglie verstanden werden – der Impuls bei jedem x ist die Wellenzahl oder die Änderungsrate der Phase der Wellenfunktion. Der Zustand (in natürlichen Einheiten wo h = 2 π ) sagt, dass die Phasenänderung, wenn Sie einer klassischen Umlaufbahn folgen, ein ganzzahliges Vielfaches von sein sollte 2 π , dh dass die Welle eine stehende Welle bilden soll.

Diese Formel ist nicht exakt, weil die Quantenwelle nicht der klassischen Trajektorie folgt, aber die WKB-Näherung nimmt dies einfach als Ausgangspunkt und erzeugt eine Welle, deren Phase durch den Wert dieses Integrals gegeben ist und deren Amplitude der ist Kehrwert der Quadratwurzel der klassischen Geschwindigkeit.

Der Grund dafür war bereits bekannt, bevor die Quantentheorie vollständig formuliert war. Aber um es zu verstehen, muss man mit Aktionswinkelvariablen vertraut sein

Aktionswinkelvariablen

Betrachten Sie eine eindimensionale Bahn eines Teilchens mit Ort x und Impuls p. Sie nennen den von der Umlaufbahn umschlossenen Bereich im Phasenraum J, und das ist die Aktion. J ist nur eine Funktion von H und zeitlich konstant (per Definition).

Die konjugierte Variable zu J ist eine Variable, die die Punkte der Umlaufbahn unterscheidet, und diese heißt θ . Jetzt stellen Sie fest, dass die Fläche im Phasenraum unter kanonischen Transformationen invariant ist (für infinitesimale kanonische Transformationen ist dies der Satz von Liouville), sodass die Fläche zwischen den Umlaufbahnen bei J und J + dJ dieselbe ist wie die Fläche in xp-Koordinaten zwischen J und J+dJ, was nur dJ ist, weil das die Definition von J ist. Aber dieser Bereich in J, θ Koordinaten ist dJ mal die Periode von θ , Also θ hat für alle J die gleiche Periode, was ich annehmen werde 2 π .

Der Kurs, zu dem θ mit der Zeit zunimmt, ist durch die Hamilton-Gleichungen gegeben

θ ˙ = H J = H ' ( J )

Und das ist über die gesamte Umlaufbahn konstant, weil H konstant ist, und J ebenfalls. Das lernt man also θ steigen monoton mit einer konstanten Rate bei jedem J und der Zeitdauer von θ ist:

T = 2 π H ' ( J )

Semiklassische Quantisierung

Angenommen, Sie koppeln dieses eindimensionale System schwach an den Elektromagnetismus. Die klassische Orbitalfrequenz ist die Frequenz der emittierten Photonen (und das Doppelte dieser Frequenz und das Dreifache dieser Frequenz). Wenn Sie also diskrete Photonenemissionsübergänge haben möchten, müssen Sie sicherstellen, dass ein Photon der Frequenz emittiert wird f = 1 T , und Energie wegnehmen h f hinterlässt einen Quantenzustand, in den man fallen kann. Wenn es also einen Quantenzustand gibt, der einer klassischen Bewegung mit einem Wert von J entspricht, muss es bei der Energie H(J) einen anderen Quantenzustand mit Energie geben

H ( J ) 2 π h T = H ( J ) H ' ( J ) h H ( J h )

mit anderen Worten, die Quantenzustände müssen gleichmäßig in J verteilt sein. In dieser Reihenfolge bedeutet dies, dass es Zustände bei Jh, J-2h, J-3h usw. gibt und Übergänge zu diesen Zuständen die klassischen Strahlungsharmonischen reproduzieren müssen erzeugt, wenn Sie das Ding schwach an Elektromagnetismus koppeln.

Die Quantisierungsregel lautet also J = n h , bis auf einen möglichen Versatz. Die Ableitung macht deutlich, dass sie nur für führende Ordnung in h gilt. Dies war Bohrs Korrespondenzargument für die Quantisierungsbedingung.

Wenn Sie mehr als einen Freiheitsgrad haben und das System integrierbar ist, haben Sie Aktionsvariablen J 1 , J 2 . . . J n und konjugieren Winkelvariablen periodisch mit Periode 2 π jeder. Sie können jeden der Freiheitsgrade schwach an den Elektromagnetismus koppeln, und jede klassische Periode des θ zeitlich variabel ist

T k = H J k

Die Aussage lautet also, dass für jeden Orbit jede J-Variable gemäß der Bohr-Regel quantisiert wird.

J k = n h

Das J k Variable ist die Fläche, die in der eindimensionalen Projektion der Bewegung in den Koordinaten eingeschlossen ist, wo sich die Bewegung in eine multiperiodische Bewegung aufteilt (dies ist der Torus von Bar Moshes Antwort). Dies ist Sommerfelds Erweiterung der Bohr-Quantisierung.

Also das Integral p d q Mit p und q wird jede konjugierte Variable genommen, die eine periodische Bewegung macht. In 1d gibt es nichts zu tun, in mehreren Dimensionen wählen Sie einfach Variablen aus, die separat eine 1d-Bewegung ausführen, und im Allgemeinen müssen Sie J-Variablen finden. Dieses Verfahren funktioniert nicht für klassisch chaotische Systeme.

Tolle Antwort @Ron. Können Sie erklären, warum es eine 2 π Faktor in der ersten Gleichung des Abschnitts "Semiklassische Quantisierung"?

Viele symplektische Mannigfaltigkeiten (Phasenräume mechanischer Systeme) lassen ein Koordinatensystem zu, in dem die symplektische Zweierform lokal geschrieben werden kann als:

ω = ich d p ich d q ich + j d ich j d θ j

Wo p ich , q ich sind lineare Koordinaten ich j sind radiale Koordinaten und θ j sind Winkelkoordinaten.

Die Untermannigfaltigkeit, parametrisiert durch θ j ist ein Torus und ein Ergebnis von Snyatycki besagt, dass das System mit Hilfe eines Hilbert-Raums von Wellenfunktionen (Verteilungen) quantisiert werden kann, der nur auf Punkten unterstützt wird, deren Koordinaten ich j erfüllen:

2 ich j = m

Eines der einfachsten Beispiele, das eine solche Quantisierung zulässt, ist die Zwei-Sphäre, deren symplektische Form ihre Flächenform ist

EIN = r s ich n θ d θ d ϕ = d z d ϕ

wo θ , ϕ sind die sphärischen Koordinaten und z ist die Koordinate entlang der Achse.

In diesem Fall ist die Bohr-Sommerfeld-Bedingung gegeben durch:

z = m 2 ,

das ist die Spinprojektionsquantisierungsbedingung.

In einer anspruchsvolleren mathematischen Sprache sagt man, dass eine offene, dichte Teilmenge der symplektischen Mannigfaltigkeit von Lagrange-Tori geschichtet ist und die Integration über die Erzeugungszyklen der Tori erfolgt.

Danke für die Antwort. Ich muss leider noch eine ganze Weile daran kauen, um es zu verstehen, da ich keine Ahnung habe, was die symplektische Zweierform ist und was das Hutprodukt macht. Außerdem höre ich zum ersten Mal von Snyatycki-Staaten.
@ user9886: Die symplektische Struktur des Kotangensbündels (dh des Impulsraums) ist für die verantwortlich in Hamiltons Gleichungen (und auch in den Poisson-Klammern); Die Hamilton-Gleichungen nehmen jedoch nur dann ihre einfache Form an, wenn Sie kanonische Koordinaten verwenden - wenn Sie beliebige Koordinatentransformationen zulassen, erhalten Sie komplexere Koeffizienten - insbesondere die Komponenten eines antisymmetrischen Tensors vom Rang 2, der symplektischen Form; Tatsächlich reicht eine symplektische Form auf einer beliebigen Mannigfaltigkeit aus, um die Hamiltonsche Mechanik durchzuführen (z. B. können Sie die Vektorbündelstruktur vergessen).